- Wed Jun 29, 2016 8:59 am
#26695
Complete question explanation.
Flaw Question. Cause and Effect. The correct answer choice is (A).
The stimulus tells us that the sample size used in this 2005 study was too small to generate valid inferences, but then concludes that because younger participants had lower levels of toxic chemicals in their system, the ban of these chemicals was effective at reducing human exposure. The stimulus identifies the banning of PCB chemicals as the cause for reduced levels of PCB exposure (the effect of the ban).
There are likely a lot of possible cause and effect flaws that could be present here, but the most glaring flaw in the reasoning is that the author initially says this sample size was too small to draw inferences from, but concludes by doing just that. That is the answer choice we should be looking for.
Answer choice (A) - This is the correct answer. This mirrors the answer choice option we prephrased.
Answer choice (B) - The argument might have overlooked this, but that's not the flaw in the reasoning. Even if the argument didn't overlook this factor, and PCBs were the only chemicals that could be involved, the reasoning would be flawed, so this cannot be the correct answer choice.
Answer choice (C) - the level of PCBs in younger participants is support for the inference that people have reduced exposure following the ban. The issue is that this is weak support, since the sample size of the study was too small.
Answer choice (D) - To take this out of abstract terms and apply it, this answer choice would mean that the argument assumed that the PCB chemical ban was the cause of the reduced human exposure to PCB, when in reality reduced exposure to PCB chemicals could have been the cause of the ban. This is irrational, especially since the stimulus set up the timeline of the ban occuring in the 1970s and the study occuring in 2005, eliminating this possibility.
Answer choice (E) - the argument doesn't explicitly address this, but similar to answer choice (B), even if this was true the reasoning of the argument would still be flawed, so this answer choice is incorrect.
Flaw Question. Cause and Effect. The correct answer choice is (A).
The stimulus tells us that the sample size used in this 2005 study was too small to generate valid inferences, but then concludes that because younger participants had lower levels of toxic chemicals in their system, the ban of these chemicals was effective at reducing human exposure. The stimulus identifies the banning of PCB chemicals as the cause for reduced levels of PCB exposure (the effect of the ban).
There are likely a lot of possible cause and effect flaws that could be present here, but the most glaring flaw in the reasoning is that the author initially says this sample size was too small to draw inferences from, but concludes by doing just that. That is the answer choice we should be looking for.
Answer choice (A) - This is the correct answer. This mirrors the answer choice option we prephrased.
Answer choice (B) - The argument might have overlooked this, but that's not the flaw in the reasoning. Even if the argument didn't overlook this factor, and PCBs were the only chemicals that could be involved, the reasoning would be flawed, so this cannot be the correct answer choice.
Answer choice (C) - the level of PCBs in younger participants is support for the inference that people have reduced exposure following the ban. The issue is that this is weak support, since the sample size of the study was too small.
Answer choice (D) - To take this out of abstract terms and apply it, this answer choice would mean that the argument assumed that the PCB chemical ban was the cause of the reduced human exposure to PCB, when in reality reduced exposure to PCB chemicals could have been the cause of the ban. This is irrational, especially since the stimulus set up the timeline of the ban occuring in the 1970s and the study occuring in 2005, eliminating this possibility.
Answer choice (E) - the argument doesn't explicitly address this, but similar to answer choice (B), even if this was true the reasoning of the argument would still be flawed, so this answer choice is incorrect.